Categories
Chicago Economists Harvard

Chicago. Milton Friedman visits the Harvard Young Conservative Club, 1964

 

At the time of Milton Friedman’s talk at Harvard, reported below in the Harvard Crimson, the 1964 Republican Presidential primaries and conventions were running hot and Senator Barry Goldwater was taking a lot of flak for his opposition to the Civil Rights Act of 1964. Milton Friedman can be seen here flying wingman for Goldwater on the issue.

________________________

Friedman Cautions Against Rights Bill
The Harvard Crimson, May 5, 1964

Milton Friedman, professor of economics at the University of Chicago and bogeyman of Ec 1, last night defended the “free-market principles” of “unanimity without conformity” against encroachments by the “coercive mechanism” of “the political method.”

In a talk sponsored by the Young Conservative Club, Friedman spent most of the evening criticising the Civil Rights Bill. “The majority in this country are prejudiced,” he stated, “and it is naive–no, it’s undemocratic,–to suppose you’re going to get people to vote against themselves.”

But he also found time to consider the tax cut (“naive”), legislation guaranteeing equal wages to women (“antifeminist”) the Federal Reserve Board (“it has never worked”), the draft (“an invasion of privacy”), legislation in general (“in case after case, laws have had the opposite effect of what was intended”), and the market mechanism (“protects the interests of minoriy groups”).

The Civil Rights Bill, said Friedman, is “wrong in principle,” because it attempts to make people “conform to the values of the majority.”

This bill is made worse, he said, because in actually there is only the “appearance of a majority” in favor of passing it. “The only reason the bill has a ghost of a chance,” he said, is that Northerners will vote for it thinking it applies to the “regional problem” of the South.

“It is extraordinary to see how naive one can be in this area” of legislation, he declared. “If we pass a law saying that race shall not be a factor in employment, then what grounds do we have for opposing a law that race shall be a factor?”

The most valid grounds, he continued, are “the general principle that the state shall not interfere in these matters.”

“The Negro is undoubtedly hurt” by segregation, said Friedman, and “the appropriate recourse is to try to persuade people that they are wrong.”

However, “the most important” solution is to eliminate “barriers” to equality, specifically, fair employment practices legislation. If the free market is allowed to operate, said Friedman, prejudice will result in lower wages for Negroes.

“Each of us separately,” he said, can then “try to offset the actions of others through our own economic activity.” By being unprejudiced and hiring Negroes, “we get things at less cost,” he said. “Not only does virtue triumph–it is even rewarded.”

SourceThe Harvard Crimson Archive.

Image Source: University of Chicago Photographic Archive, apf1-06231, Hanna Holborn Gray Special Collections Research Center, University of Chicago Library.

Categories
Chicago Economics Programs Faculty Regulations

Chicago. Economics A.M. requirements amended to become “Consolation Prize”. Lewis and Schultz, 1950

 

In 1950 the Chicago economics department voted to convert its master’s degree into an award for the successful partial completion of its Ph.D. program. It was to serve as a “consolation prize” for good graduate students but those found not to have the right “Ph.D. stuff” (H. Gregg Lewis’ words in his memo of Sept. 29, 1950 to chairman T. W. Schultz, transcribed below). I have also included the relevant portion of the distributional and examination requirements for the Ph.D. that had already formed part of the so-called “alternate departmental master’s degree”. H. Gregg Lewis’ proposal was largely accepted by the department (minutes from the meeting of November 2, 1950 transcribed below), thereby eliminating distinct tracks for its A.M. and Ph.D. degree programs, respectively.

_______________________

ALTERNATIVE DEPARTMENTAL MASTER’S DEGREE
[1950-51 regulations]

Upon request the Department will consider recommending for the Master’s degree candidates who have satisfied the distribution requirement for the Ph.D. degree and have passed with satisfactory standing the three written field examinations for the Ph.D. degree. One modern foreign language is required. In place of a thesis such candidates may present an acceptable paper or report on a problem approved by the Department.

[…]

Distributional requirement [Ph.D.]. The candidate is expected to have familiarity with the subject matter equivalent to that covered in at least one course (200 or 300 level of reasonable comprehensivenss in each of ten fields (five required and five elected), satisfactory evidence of which can be provided by course credit or by passing a special examination. The required fields are: (a) economic theory, (b) accounting, (c) statistics, (d) economic history, and (e) money, banking, and monetary policy. The fields from which five may be elected are: (f) consumption economics, (g) industrial relations, (h) monopoly and public utilities, (i) agricultural economics, (j) government finance, (k) international economic relations, and (l) substitute fields, but not in excess of two, proposed by the candidate and approved by the Departmental counselor or the Department. One or both of these substitute fields may be outside the Department of Economics, and in general some work outside the Department is recommended with a view to rounding out a program appropriate for the individual student. In case of students transferring from other institutions, adequate training in general history may be substituted for economic history upon the written recommendation of the Departmental counselor.

Preliminary written field examinations [Ph.D.]. In each of three fields of specialization, in addition to presenting course credit or special examinations to show satisfactory preparation, the candidate will be required to pass a written examination.

The candidate is expected to select the three fields of specialization—a primary field and two secondary fields—for intensive graduate work. The primary field is that in which the [Ph.D.] thesis will be written. One of the three fields (primary or secondary) must be that of economic theory, including monetary theory. The fields from which selection is to be made are listed above under the heading “Distributional Requirement,” except that accounting may not be chosen as a field without approval of the Department. One secondary field of specialization may be a field named by the candidate outside the list above, and this may be in a department other than Economics. A secondary field may also be developed under one of the interdepartmental committees of specialization International Relations, Human Development, Planning or Social Thought. The program of work proposed, which ordinarily will include four to five courses, must be approved by the Department. No other secondary field may replace the required field in economic theory.* Students should consult with the Departmental counselor with respect to appropriate programs of work in preparation for the field examinations. The field examinations are given by the Department in the sixth and seventh weeks of the Winter and summer quarters. Application for any field examination should be made not later than the end of the first week of the quarter in which the examination is to be taken.

*Students who take the field examination in money, banking, and monetary policy will not be required to write the monetary theory part of the economic theory examination.

 

Source: University of Chicago, Announcements. The Division of the Social Sciences, Sessions of 1950-1951, Vol. L, Number 9 (July 20, 1950), pp. 25-26.

_______________________

ECONOMIC COURSES LISTED IN THE LEWIS MEMO (29 Sept 1950) AND INCLUDED IN THE DEPARTMENTAL MINUTES (2 Nov 1950)

209. Intermediate Economic Theory. (Procter Thomson/Harold Gregg Lewis) Designed for students majoring in economics. Deals with factors controlling production, value and relative prices, and distribution.

211. Introduction to Statistics. (Harold Gregg Lewis) Elementary principles of statistics. Main topics: frequency distributions, averages, dispersion, index numbers, elements of the theory of statistical inference.

220. Economic History of the United States. (Earl J. Hamilton) Facts and factors in American’s economic growth from the Colonial period to World War II, including the development of agriculture, industry, commerce, finance, and transportation; economic effects of wars; role of the entrepreneur; rise in living standards; unrest and utopias in periods of stagnation; commercial crises and economic basis of cultural progress.

222. The Rise of Industrial Civilization in Europe. (John Ulrich Nef) Economic development in its relation to religious, political, intellectual and artistic history since the seventeenth century.

230. Introduction to Money and Banking. (Milton Friedman/Lloyd Wynn Mints) Factors which determine the value of money in the short and in the long run; and operation of the commercial banking system and in relation to the price level and general business activity.

240. Introduction to Industrial Relations. (Albert E. Rees) The nature of the labor market; government regulation of wages; social security; the history, structure, and functions of American labor unions; and collective bargaining. Special attention is given to current problems of public policy.

255. Introduction to Agricultural Economics. (D. Gale Johnson). Nature of resources used in agriculture. Prices, production, resource allocation, and income distribution. Analysis of government programs, subsidies, storages, crop control, soil conservation, food-stamp plan.

260. Introduction to Government Finance. (Richard B. Goode) Survey of institutions and theories of government finance. Effects of public expenditures; functions of public revenue; forms of taxation; tax criteria; determination of tax policy; public borrowing; debt management; fiscal policy.

270. International Economics. (Bert F. Hoselitz) The nature of international payments and receipts; foreign trade and banking system. The gold standard in the interwar period. The breakdown of the gold standard and the period of fluctuating exchange rates. Exchange controls, clearing agreements and payments agreements. The second world war and the foreign exchange markets. The position of the International Monetary Fund and the International Bank for Reconstruction and Development in the present world economy.

271. Economic Aspects of International Politics. (Bert F. Hoselitz) An introductory survey, with particular reference to the United States, of the economic policies and activities of governments. Topics: international specialization of production and the distribution of world resources, structure of international exchanges and the mechanism of international transfer of goods and services; tariffs and other regulatory measures; trade agreements and the most-favored nation clause; international flow of capital and investment; the position of the ITO, the IMF, the ECA and other official agencies in international trade and exchange.

300A, 300B. Price Theory. (W. Allen Wallis 300A/Lloyd A. Metzler 300B/Milton Friedman 300B) A systematic study of the pricing of final products and factors of production under essentially stationary conditions. Covers both perfect competition and such imperfectly competitive conditions as monopolistic competition, oligopoly, and monopoly. 300A deals primarily with the pricing of final products; 300B, with the pricing of factors of production.

Source: University of Chicago, Announcements. The Division of the Social Sciences, Sessions of 1950-1951, Vol. L, Number 9 (July 20, 1950), pp. 27-28.

_______________________

THE UNIVERSITY OF CHICAGO

September 29, 1950

To        T. W. Schultz

From   H.G. Lewis

In Re: Requirements for the Master’s Degree

This is an elaboration of comments made to you this summer concerning the Master’s degree.

I should like to recommend the following changes in our requirements for the A.M. degree:

  1. That the distinction between the “regular” A.M. and the “alternative” A.M. be abolished.
  2. That the departmental requirements for the A.M degree consist of the following alteration of the present alternative A.M. requirements:
    1. A distribution requirement covering five (rather than the present eight) fields of economics of which theory, statistics, and money and banking shall be mandatory. Students who do not hold the traditional A.B. degree must meet the requirement by passing satisfactorily a qualifying examination coving the subject matter of Economics 209, 211 (unless the student has passed the Divisional qualifying examination), 230, and two courses chose from 220, 222, 240, 255, 260, 270, 271.1/ Students holding the traditional A.B. may meet the requirement by showing equivalent course credit.
    2. The passing of two Ph.D. field examinations (with Part I of the Theory exam counting as a full exam) at a satisfactory level (that is, at either the Ph.D. level or at a level somewhat lower but not so low as not to warrant blessing the candidate with a Master’s degree).2/
    3. A showing of competence in economic principles; made either by passing (at the A.M. level or higher) Part I of the Theory examination, by course credits or course examinations in Economics 300A and 300B, or by equivalent course credit.

I would recommend that the changes in requirements become effective as of the beginning of the Summer Quarter, 1951 for students entering the Department in that and later quarters.

1/ This qualifying examination is now offered every quarter. This is an extravagant use of faculty. I should like to see the exam offered only once a year. Furthermore, I should like to permit students to substitute course grades for all or part of the exam provided the course grades are for courses taken here and provided they are not at a level lower than B.

2/ There would be therefore no special examinations for A.M.’s, but the examinations would be graded into three levels: passing for the Ph.D. and A.M. degrees, passing for the A.M. degree but not for the Ph.D., failing for both.

I would urge students to give requirement (b) high priority in preparing their programs of study.

Since the ends sought by these changes can be reached in other ways, I specify below what these ends are.

I view our principal instructional purpose as that of training high-level (Ph.D. and beyond) professional economists. I think we ought to view our training of “junior” economists and the awarding of the A.M. degree only as an incident arising from the fact that at the time a student applies for admission to the department, we cannot predict accurately either his calibre as a student or his academic goals.

It seems to me that the requirements for the Master’s degree should meet these tests:

  1. They should include no requirements which the Department would not make for the Ph.D. degree. Otherwise both student and faculty time will be spent in activities extraneous to the training of high-level economists. The present alternative A.M. meets this test but the regular A.M. does not.
  2. The requirements should be at a level high enough to be respected by the academic world. Both present degrees meet this test, I believe.
  3. But the standards for the degree should not be so high that potentially able Ph.D. candidates will be deterred from entering because of the considerable risk that if they fail to meet Ph.D. standards they will also fail to meet A.M. standards. If we set standards for the A.M. that are almost as likely not to be met as the Ph.D. standards we hold out no “consolation prize” to those good students who are fearful of not being able to meet Ph.D. standards. The present alternative A.M. requirements do not meet this test.

One of the ways by which we can raise the calibre of our Ph.D. candidates without reducing our enrollment is to increase the number of students who are given an opportunity to show at close hand their potentialities to us and to screen out at an earlier date those who are not of Ph.D. stuff. I am confident that quite accurate screening can take place ordinarily by the end of the first year of graduate residence. I contemplate our using the A.M. requirements as a screening device; the present alternative A.M. is not satisfactory from that point of view since it postpones too long the screening decision.

Source: University of Chicago Archives. Department of Economics Records. Box 41, Folder “41.8”.

_______________________

MINUTES
Meeting of the Department

Time and Place: Thursday, November 2, 1950, at 1:00 p.m. in Room 424, Social Science Research Building.

Present: T. W. Schultz (chairman), H. G. Lewis, A. Rees, R. Goode, G. Tolley, D. G. Johnson, F. H. Harbison, J. Marschak, C. Hildreth, F. Knight, L. Metzler.

  1. Handling of Student Business
    It was agreed that all bona fide applications for admission to candidacy filed this quarter would be considered as falling under present degree requirements even though Departmental action does not take place until Winter quarter.
  2. Ford Foundation
    Schultz stated that as a Department we have an obligation to ourselves, to the University, and to the community more largely to think through carefully the problem of making the best use of the Ford Foundation’s present grant of $300,000 to the University as well as possible later grants. There was a brief general discussion of the problem.
  3. Departmental Rules Governing Residence and Availability to Students
    Schultz pointed out that in the current year we have been able for the first time to reduce direct teaching loads for most of our members to four courses per year or less. This reduction, he pointed out, makes it desirable that the Department impose upon itself rules governing residence and availability to students and others in the university community lest they be imposed upon us from outside. The problem of rules for residence involves not only a rule stipulating in some way minimum residence, but also the question of whether “free” quarters out of residence should be considered a matter of a right accruing to an individual from his residence or a privilege dependent upon ad hoc decisions made by the Department chairman and the Dean. Schultz expressed himself as being in favor of a rule somewhat similar to the rules for accumulating sabbatical leave under a 3Q contract. In addition there is the problem of insuring, perhaps by rule, “availability” when in residence. The formulation of appropriate rules is to come before the Department for its consideration in the Winter quarter.
  4. The Department considered Lewis’ recommendations for changes in the A.M. requirements. (See attached memo. [above]) the following amendment of Lewis’ recommendation was passed:
    1. That the distinction between the regular A.M. and the “alternative” A.M. degrees be abolished.
    2. That the Departmental requirements for the A.M. degree consist of the following:
      1. A distribution requirement to be met by passing a “Qualifying” examination covering the subject matter of Economics 209, 211 (unless the student has passed the Divisional qualifying examination) 220 or 222, 230 and two courses chosen from 240, 255, 260, 270, and 271. Students holding the traditional A.B. may satisfy the requirement by equivalent course credit.
      2. The passing of two Ph.D. field examinations (with Part I of the Theory examination counting as an examination) at a satisfactory level of A.M. candidates.
      3. A showing of competence in economic principles; made either by (at the A.M. level or higher) Part I of the Theory examination, by course credits or examinations in Economics 300A and 300B, or by equivalent course credit.
      4. An acceptable paper or report on a problem approved by the Department. The paper will be read by two members of the Department of which the course instructor will be one in the event the student submits a term paper prepared for a course.

The above changes in requirements are to become effective as of the beginning of the Summer quarter, 1951 for students entering the Department in that and later quarters.

It was understood that the above motion in no way changes present preliminary examinations or other requirements for the Ph.D. degree. Professor Knight asked the minutes to show his objection to dropping Economics 210 (Accounting) from the requirements for the A.M. degree.

  1. Student Business
    1. Petitions

Lawrence Bostow’s petition for approval of French and Russian as languages for the Ph.D. was approved.

Mr. H. M. Herlihy’s petition for the field of “Social Organization” (Sociology Department) as his third field for the Ph.D. degree was approved.

Mr. John Holsen’s petition for a third Ph.D. field in Planning (Planning Department) was approved. Mr. Johnson, his counselor, was asked to inform Mr. Holsen, however, that this approval does not entitle Holsen to shorten his total program in Economics for the Ph.D.

Mr. Edward Mishan’s petition for approval of Spanish as a Ph.D. language was denied.

    1. Admission to Candidacy

Mr. Howard Ammerman’s application for admission and for approval of a thesis topic was moved to the bottom of the list of applications.

Mr. Rondo E. Cameron’s application for admission to candidacy for the Ph.D. degree was recommended to the Division for approval, contingent upon his passing the Theory examination (written Summer, 1950) and his proposed thesis topic, “French Foreign Investment, 1815-1870,” was approved. Thesis committee: E. J. Hamilton, chairman, L. Metzler, P. Thomson.

After some discussion, Mr. Clifford Clark’s application was moved down the list. Lewis was instructed to advise Clark to consult with Hamilton concerning the latter’s misgivings about the proposed thesis topic, and in addition to confer with Hayek, Knight, and other members of the Department concerning the thesis topic.

Mr. George P. Coutsoumaris’ application for admission to candidacy for the Ph.D. degree was recommended to the Division for approval, contingent on his passing the Theory examination (written Summer, 1950), and his proposed thesis topic, “Possibilities of Increasing Economic Efficiency in Greek Agriculture,” was given qualified approval, the Department suggesting that he limit the topic somewhat preferably to a topic approximately the same as that covered in the sections (VII and VIII) of his outline dealing with capital in Greek agriculture. Thesis committee: D. G. Johnson, chairman, C. Harris, J. Margolis (planning).

Mr. David Fand’s proposed thesis topic, “Monetary Theory of the Federal Reserve Board,” was discussed. It was agreed to come back to it at the next meeting after several more of the members of the Department had an opportunity to discuss the topic with Fand.

The meeting was adjourned at 3:05 p.m.

Source: University of Chicago Archives. Department of Economics Records. Box 41, Folder “41.8”.

Images:  University of Chicago Photographic Archive, H. Gregg Lewis [apf1-03861] and T. W. Schultz [apf1-07479], Hanna Holborn Gray Special Collections Research Center, University of Chicago Library.

Categories
Economists Wisconsin

Wisconsin. Debate on role of federal government in the economy. Keyserling vs. Friedman, 1963

Rummaging through the digitized archival offerings of the University of Wisconsin library, I came across the poster above announcing a debate betwen Leon Keyserling and Milton Friedman. The poster gives a date and time for the debate but no year was included for the obvious reason that advertising years in advance on posters seldom makes sense. The webpage where the image of the poster is displayed is hardly helpful in dating the debate:

Economic debate poster

  • ca. 1961-ca. 1973
  • A poster announcing a debate on government action on the economy to be held at the Union Theater. The poster once hung in Van Hise Hall and was found during construction.

From the following two articles in the Wisconsin State Journal we find that the debate took place in 1963. It is also somewhat interesting to note that the debate took place just eight days before the assassination of John F. Kennedy in Dallas, Texas. 

________________________

U.S. Government Role in Economy Will Be Debated

Economists Leon Keyserling and Milton Friedman will debate the proposition that the federal government should play an active roll in the economy, at 8:15 tonight in the Wisconsin Union theater.

Keyserling, chairman of the Council of Economic Advisers to the President from 1950 until the end of the Truman Administration, will present the affirmative aide of the argument.

He is presently the head of the Conference on Economic Progress and a consulting economist and attorney in Washington. He has served as deputy administrator of the U.S. Housing Authority, and as consultant to various committees and members of the Senate and the House.

Keyserling has been connected with the studies and drafting of such economic legislation as the National Industrial Recovery act, the National Housing act, the National Labor Relations act, the Employment act, and the General Housing act.

Debating the negative side of the proposition will be Friedman, professor of economics at the University of Chicago and a member of the research staff of the National Board [sic] of Economic Research.

A visiting professor at the University of Wisconsin in 1940-41, Friedman has also served as an associate economist with the National Resources committee and as the principal economist for the division of tax resources of the U.S. Treasury Department.

The debate is sponsored by the Wisconsin Union Forum committee and the Pan-Hellenic and Interfraternity Associations. No tickets are needed.

Source: Wisconsin State Journal. Madison. Thursday, November 14, 1963, p. 4.

________________________

1,000 Hear Top Economists Debate Role of Government
By William Hauda (State Journal Staff Writer)

Economists Leon Keyserling and Milton Friedman Thursday night didn’t reach any agreement on what role the federal government should play in the national economy.

Keyserling, a liberal and a leading consulting economist and attorney in Washington, D. C, contended that the federal government should play an active role in the economy.

Debate Before 1,000

Friedman, a conservative and professor of economics at the University of Chicago, argued the negative in a debate before more than 1,000 persons in the Wisconsin Union theater.

Keyserling maintained that today’s world problems of competing economic systems and large armaments “can only be solved through the system we call government”

“These things will continue to require sensitive, progressive, alert, and — yes — expanding public responsibility,” he said.

Friedman said, “Of course we all know the federal government has for a long time played a role in the economy. I am sure that; Mr. Keyserling, like all of us, would like to see a world where less is spent on armaments.”

He went on to argue that, in most every case, federal control has not accomplished its objectives.

He said it was “not a simple issue of responsible government” and that he wanted to see a system in which a producer can’t get government protection and can only prosper by serving the consumer.

Cites Travels

Friedman said neither the Interstate Commerce Commission (ICC) nor the Federal Communications Commission (FCC) has accomplished their objectives and described anti-trust legislation as a “mixed bag” of good and bad.

Of his recent travels around the world, he said, “What struck me was wherever there was extensive central planning, there was poverty.” As examples, he cited Russia, India and Indonesia.

“I am not saying free enterprise will produce prosperity,” he added. “By itself, it is not a sufficient condition.”

In his rebuttal, Keyserling said Friedman’s arguments were like saying if there is corruption in the New York police department, we should just get rid of the police.

“Next, my friend trys to tell you that the farm program has accomplished nothing. Goodness knows, there are thorns in the farm program. But what person can stand up in the state of Wisconsin and tell you the farmer is not better off than 40 years ago.”

“My friend wants to do away with the public operation of secondary schools,” said Keyserling. I don’t. I think we need more schools.”

“I’m the first one to the government has not perfected the degree of control of business cycles that I would like to see it obtain,” said Keyserling. But he added, “all this progress has been made within our life time and I’d like to see it continue.”

Friedman said, “The best thing the government can do is provide a stable (economic) background.” He said the government should not disrupt the economy and should allow free enterprise to run the economy.

Source: Wisconsin State Journal. Madison. Saturday, November 16, 1963, p. 7.

Image Source: University of Wisconsin digital library.

Categories
Agricultural Economics Chicago Iowa Minnesota Suggested Reading Syllabus

Minnesota. Course outline and reading for graduate macroeconomics. Brownlee, probably 1959

 

Based on a pamphlet in which he argued that “properly fortified margarine ‘compared favorably’ with butter in nutrition and palatability”, the economics Ph.D. student, Oswald Harvey Brownlee (1917-1985), brought the wrath of the Iowa Farm Bureau among others down upon himself and his economist seniors. After the President of Iowa State caved to the state’s dairy interests in the matter, Theodore  W. Schultz, D. Gale Johnson, and O. H. Brownlee were all to ultimately head off to the University of Chicago.

Oswald Harvey Brownlee. Putting dairying on a war footing, 64 page pamphlet published by Iowa State College Press, 1944.

See: Seim, David L. “The Butter-Margarine Controversy and “Two Cultures” at Iowa State
College.” The Annals of Iowa 67 (2008), 1-50.

Also mentioned in: Milton and Rose Friedman, Two Lucky People: Memoirs, p. 193.

Brownlee went on to teach at the University of Minnesota, where we found him teaching a graduate macroeconomics course. Clearly that was still time that the hatches separating microeconomics and macroeconomics were not so securely battened as today. “Public finance” was Brownlee’s major field so his broad fiscal policy interests make sense.

The course outline transcribed in this post comes from Martin Bronfenbrenner’s papers at the Economists’ Papers Archive at Duke University. Bronfenbrenner taught at the University of Minnesota from 1959-1962 and we can presume that the copy of Brownlee’s macroeconomics course outline with readings was for either 1958-59 or 1959-60.  A second, apparently later, version of the course outline for “Economics 176A” with “Brownlee” handwritten in the upper right corner is also found in the same folder. Three new readings from the second copy have been added and placed within square brackets below. The readings in Parts I and II, IX and X were not included in the second outline for “Economics 176A”.

_______________________

Handwritten note at top:
“Martin, Here is the outline for the Macro theory. Which part do you want to teach? [signed] Oz”

 

Economics 176A-B
Course Outline and Suggested Readings

This brief outline and reading list is intended to serve as a general summary of the materials to be considered during the course and as a guide to class discussion and to outside reading. The detail in the outline does not necessarily correspond to the detail in class discussion. The most significant readings are starred (*). The literature in this field has grown so rapidly during the past decade that this reading list cannot be considered as a complete bibliography of relevant writings.

It is hoped that during the quarter the student will gain an adequate understanding of how the equilibrium values of the relevant variables (gross national product, employment, the general level of prices and the rate of interest, for example) might be determined, and how changes in certain exogeneous variables (including various economic policy variables) might affect these equilibrium values. Although the primary emphasis of the course is on equilibrium levels of certain variables, an introduction to dynamic analysis (a description of the path of a variable over time) will be offered. This will provide the basis for subsequent discussion of business cycle theory and growth models.

  1. General Orientation of the Course
    1. Relationship of macro-static theories to other classes of economic theories
    2. Limitations of macro-static analysis as a basis for policy statements
  2. The firm’s Demand for Labor
    1. Importance for labor hired by business firms in the labor market as a whole
    2. Static theory of production with emphasis on the demand for labor.
      1. Nature of the firm’s production function
      2. Determinants of equilibrium level of employment within the firm
      3. Comparisons of equilibrium levels of employment under various resource market, product market and technological conditions
    3. Effects of Changes in Quantities of Other Resources Upon Demand for Labor

Readings:

1—K. E. Boulding, Economic Analysis, Chapter 31 (revised edition)

2—George Stigler, The Theory of Price, Chapters 6-11.

3—Paul A. Samuelson, Foundations of Economic Analysis, Chapter 3, pp. 21-33.

4—Joan Robinson, Economics of Imperfect Competition, Books VII and VIII.

  1. Equilibrium in the Labor Market for the Economy as a whole
    1. Aggregation of outputs, labor inputs, wage rates and prices
    2. Determination of various combinations of general level of prices and “real” output which will maintain equilibrium in the labor market—an “aggregate supply” function.
      1. With money wage rate autonomously determined: a wage “floor”, a wage “ceiling”, both a “floor” and a “ceiling”.
      2. With supply of labor dependent upon “real” wages.
      3. With supply of labor dependent upon “real” and money wages: the effects of asset holdings.
    3. Degree of Determinateness of relevant variables given only equilibrium in the labor market.
      1. Price level, real output and employment not uniquely determined
        1. Various combinations of price level and real output will maintain equilibrium in labor market, given the autonomously specified money wage or given fixed monetary debts and credits and flexible money wages.
        2. Employment is determined only upon the real wage, real output and employment are uniquely determined, but price level is not.

Readings:

1.*—Jacob Marschak, Income, Employment and the Price Level, Lectures 19 and 20.

2.—Sidney Weintraub, Income and Employment Analysis, Chapters 11 and 13.

3.—Francis M. Boddy, et al., Applied Economic Analysis, pp. 229-248.

4.—O. H. Brownlee, Economics of Public Finance, pp. 47-51.

5.—Don Patinkin, Money, Interest and Prices, IX-XII.

6.—Louis Hough, “An Asset Influence in the Labor Market”, Journal of Political Economy, June 1955.

7.—Robert Solow, “Technical Change and the Aggregate Production Function”, Review of Economics and Statistics, August 1957.

[8.*—Gershon Cooper, “Taxation and Incentive in Mobilization” in Readings in Taxation edited by Musgrave and Shoup.]

  1. Aggregate Demand for Goods and Services: The “Crude Classical Theory”
    1. The Quantity Identity
      1. The Demand for Money—a linear function of money income (expenditure)
      2. Assuming the supply of money (M) and the fraction of income which people with to hold as cash balances are independently determined, the equilibrium level of total money expenditure is determined.
      3. Effects of changes in money demand and money supply upon equilibrium level of money income or expenditure.
      4. Incorporation of assets as a variable influencing the demand for money
      5. Information obscured by the simple quantity identity (that omitting assets as a variable)
        (Note: further analysis of the quantity identity in terms of the kind of aggregate demand function for goods and services which it might imply will be made in subsequent sections).
    2. Equilibrium in the Labor, Money, and Commodity Markets under the assumption of the quantity identity.
      1. Quantity of labor supplied a function only of money wages
      2. Quantity of labor supplied a function only of “real” wages
      3. Division of “real” output between consumption and investment.

Readings:

1.*—J. M. Keynes, The General Theory of Employment, Interest, and Money, Chapters 2 and 19

2.—L. Klein, The Keynesian Revolution, Chapter 1 and the technical appendix, pp. 199-205

3.—Albert G. Hart, Money, Debt and Economic Activity, Chapters IV-VI and VIII

4.—Alvin Hansen, Monetary Theory and Fiscal Policy, Chapters 1-3

5.—Franco Modigliani, “Liquidity Preference and the Theory of Interest and Money”, Econometrica, 12: 45-88 (January, 1944)

6.—Seymour Harris, (editor) The New Economics, Part IX, Chapter XLI

7.—Francis M. Boddy, et al., Applied Economic Analysis, Chapter 12, 13 (pp. 222-229)

8.*—Jacob Marschak, Income, Employment and the Price Level, Lecture 2.

9.—Don Patinkin, Money, Interest and Prices, I-VIII

10.—Archibald and Lipsey, “Monetary and Value Theory,” Review of Economic Studies, October, 1958

11.*—Milton Friedman, Studies in the Quantity Theory of Money, Chapter I

12.—James Tobin, “The Interest-Elasticity of Transactions Demand for Cash”, Review of Economics and Statistics, August, 1956

13.—H. Rose, “Liquidity Preference and Loanable Funds,” Review of Economic Studies, XXIV (1956-57)

14.—Don Patinkin, Liquidity Preference and Loanable Funds, Economica, November, 1958

15.—Vera Lutz, “Multiplier and Velocity Analysis: A Marriage”, Economica, February, 1955

16.—G. C. Archibald, “Multiplier and Velocity Analysis: An Amendment”, Economica, August 1956

[17.—Ira O. Scott, “The Availability Doctrine: Theoretical Underpinnings”, Review of Economic Studies, XXV No. 1, 41-48]

  1. Aggregate Demand for Goods and Services: The “Keynesian Theory”
    1. Equilibrium in the “Commodity Market”
      1. Consumption (and Saving)
        1. Relationship to income
        2. Relationship to rate of interest
      2. Investment
        1. Relationship to the rate of interest
          1. The marginal efficiency of capital
          2. Uncertainty and the level of investment
        2. Relationship to current income
      3. The Equating of Savings and Investment (Aggregate Demand for Commodities = Aggregate Supply of Commodities)
      4. Determination of various combinations of the rate of interest and real income which will fulfill the condition for equilibrium in the commodity market (will make savings = investment)
    2. Equilibrium in the Money Market
      1. The Liquidity Preference Schedule (The Demand for Money)
      2. With money supply (M) autonomously determined, there will be various combinations of the rate of interest, real output and the price level which will provide for equilibrium in the money market.
        1. The general case
        2. The special “Keynesian” case
    3. Simultaneous Equilibrium in the Money and Commodity Markets: An Aggregate Demand Function
      1. Equilibrium rates of real output and price level which fulfill the conditions for equilibrium in both the money and commodity markets.

Readings:

1.*—Keynes, The General Theory of Employment, Interest, and Money

2.—The Keynesian Revolution (*particularly Chapter 3)

3.*—J.R. Hicks, “Mr. Keynes and the Classics”, Econometrica, 4: 147-159 (April, 1937); also included in Readings in Income Distribution, The Blakiston Co.

4.*—Franco Modigliani, “Liquidity Preference and the Theory of Interest and Money”, Econometrica, 12; 45-88 (January, 1944)

5.—Alvin Hansen, Monetary Theory and Fiscal Policy, Chapters 4-6

6.—Sidney Weintraub, Income and Employment Analysis, Part II

7.—K.E. Boulding, The Economics of Peace, Chapters 7-9

8.—Wassily Leontief, “Postulates; Keynes” General Theory and the Classicists”, included in The New Economics, Part 4, Chapter XIX

9.—The New Economics, Parts 3 and 9

10.—Abba P. Lerner, The Economics of Employment, Part II

11.*—Jacob Marschak, Income, Employment and the Price Level, Lectures 3-18

12.—O.H. Brownlee, “The Theory of Employment and Stabilization Policy” Journal of Political Economy, Oct. 1950, pp. 412-24.

13.—Ira O. Scott, Jr., “An Exposition of the Keynesian System”, The Review of Economic Studies, XIX, (1), pp. 12-18

14.—Joan Robinson, “The Generalization of the General Theory”, included in The Rate of Interest and Other Essays.

15.—Louis Hough, “The Price Level in Macroeconomic Models”, The American Economic Review, June, 1954, pp. 269-86.

16.—Milton Friedman and Gary S. Becker, “A Statistical illusion in Judging Keynesian Models”, Journal of Political Economy, February, 1957

17.—L. R. Klein, “The Friedman-Becker Illusion,” Journal of Political Economy, December, 1958; and Friedman & Becker, “Reply”, same issue.

18.—Martin J. Bailey, “Saving and the Rate of Interest”, Journal of Political Economy, August, 1957.

[19.—Hans Brems, Output, Employment, Capital and Growth, Part I.]

  1. The Equilibrium Levels of Output, Employment, Prices and the Rate of Interest in the Keynesian System.
    1. Aggregate Supply and Aggregate Demand with Flexible Money Wages
    2. Aggregate Supply and Aggregate Demand with Labor Supply Perfectly Elastic at a Given Money Wage
    3. Effects of Changes in Autonomous Variables and Parameters
      1. The autonomous component of investment
        1. The multiplier
      2. Government expenditure for goods and services
      3. The export surplus
      4. Money wage rates
      5. Technology
      6. The degree of monopoly and employers’ market expectations
      7. Population and the labor supply
      8. The money supply
      9. Marginal propensities to consumer and invest
  2. An alternative Macro-Static System
    1. Some weaknesses in the Keynesian theory
      1. A change in the structure of the system required to explain U.S. postwar experience
      2. Increased savings: income ratio as income increases not empirically verified.
    2. Assets consumption as a variable affecting
      1. Real Assets
      2. Monetary assets (cash and government debt)
      3. Aggregate demand for goods and services when assets are included as a variable in the consumption function
        1. Comparison with quantity theory
        2. Comparison with Keynesian theory
    3. The Duesenberry-Modigliani Hypothesis
    4. Including assets in other Functions: Labor Supply and Demand for Money

Readings:

1.*—Don Patinkin, “Price Flexibility and Full Employment”, American Economic Review, 38: 543-64 (September, 1948).

1a.*—Don Patinkin, Money, Interest and Prices, XIII-XV and appropriate appendices.

2.—__________, “The Indeterminancy of Absolute Prices in Classical Economic Theory”, Econometrica, 17: 1-27

3.—__________, “Involuntary Unemployment and the Keynesian Labor Supply Function”, Economic Journal, LIX: 360-83

4.—Haavelmo, Hickman, Leontief and Phipps on Patinkin, Econometrica 18: 1-26 (January, 1950)

5.—James Tobin, “Money Wage Rates and Employment”, included in The New Economics, Part 8, Chapter XL.

6.—Arthur Smithies, “Effective Demand and Employment”, included in The New Economics, Part I, Chapter XXXIX.

7.—A. P. Lerner, “Mr. Keynes’ General Theory of Employment, Interest, and Money”, Reprinted in The New Economics, Part 3, Chapter XI

8.*—Milton Friedman, “A Monetary and Fiscal Framework for Economic Stability”, American Economic Review, 38: 245-64 (June, 1948)

9.—A. C. Pigou, “Economic Progress in a Stable Environment”, Economica, 1947, pp. 180-90

10.—A. C. Pigou, “The Classical Stationary State”, Economic Journal, 53: 343-51 (1943)

11.*—James Duesenberry, “Income-Consumption Relations and Their Implications”, included in Income, Employment and Public Policy, Essay III in Part One, and as Chapter I in Income, Saving, and the Theory of Consumer Behavior.

[11a.—John H. Power, “Price Expectations, Money Illusion, and the Real-Balance Effect”, Journal of Political Economy, April, 1959, 1331-43.]

12.*—Franco Modigliani, “Fluctuations in the Saving-Income Ratio: A Problem in Economic Forecasting”, included in National Bureau of Economic Research, Studies in Wealth, Volume XI, pages 371-443.

13.—Paul A. Samuelson, “The Simple Mathematics of Income Determination”, included in Income Employment and Public Policy,” Essay VI in Part One.

14.—Oscar Lange, Price Flexibility and Employment, particularly Chapters I-V and IX-XI.

15.—Donald M. Fort, “A Theory of General Short-Run Equilibrium,” Econometrica, 13: 293-310 (October, 1945)

16.—Sidney Weintraub, Income and Employment Analysis, Part III

17.—G. L. Bach, “Monetary-Fiscal Policy Reconsidered”, Journal of Political Economy, LVII: 383-94 (October 1949)

18.—George Terborgh, The Bogey of Economic Maturity.

19.—A. P. Lerner, Economics of Employment, parts IV and V.

20.*—William Hamburger, “The Determinants of Aggregate Consumption”, Review of Economic Studies, XXII (1), pp. 23-34

21.*—Franco Modigliani and Richard Brumberg, “Utility Analysis and the Consumption Function”, included in Kenneth Kurihara, The Post Keynesian System—Essays in Honor of John Maynard Keynes.

22.—O. H. Brownlee, Economics of Public Finance, Chapters 3-6

23.—__________, “The Theory of Employment and Stabilization Policy”, Journal of Political Economy, October, 1950, pp. 412-24.

24.*—Milton Friedman, A Theory of the Consumption Function (particularly chapters 1-4.)

  1. Monetary-Fiscal Policy
    1. Effects of changes in government expenditures for goods and services, net tax collections, the tax structure and the supply of money on the demand for and supply of goods and services.
      1. In the Keynesian System
      2. In the Alternative System
    2. Built-In Flexibility vs. Ad. hominum [sic, “ad hoc”] changes.

Readings:

1.—Robert L. Bishop, “Alternative Expansionist Fiscal Policies: A Diagrammatic Analysis”, Lloyd A. Metzler, ed. Income, Employment and Public Policy.

2.—O. H. Brownlee, “Taxation and the Price Level in the Short Run”, The Journal of Political Economy, February, 1954, pp. 26-33.

3.—__________, The Economics of Public Finance, Chapter 6.

4.—Paul A. Samuelson, “Principles and Rules in Modern Fiscal Policy: A Neo-Classical Reformulation”, included in Money, Trade, and Economic Growth.

5.*—Milton Friedman, “the Effects of a Full-Employment Policy on Economic Stability: A Formal Analysis”, included in Essays in Positive Economics.

6.—E. Cary Brown, “The Static Theory of Automatic Fiscal Stabilization”, Journal of Political Economy, October 1955.

7.—Alfred Conrad, “The Multiplier Effects of Redistributive Public Budgets”, Review of Economics and Statistics, May, 1955.

8.—William A. Salant, “Taxes, Income Determination and the Balanced Budget Theorem”, Review of Economics and Statistics, May, 1957.

[9. Bent Hansen, The Economic Theory of Fiscal Policy.]

  1. Some Applications of Static Macroeconomic Analysis to Other Problems
    1. Disaggregated Systems
    2. Effects of Shifts in Expenditure and Income in One Sector upon Income in Other Sectors.

Readings:

1.—John S. Chipman, The Theory of Inter-Sectoral Money Flows and Income Formation.

2.—D. Gale Johnson and O. H. Brownlee, “Reducing Price Variability Confronting Primary Producers”, Journal of Farm Economics, May, 1950, 176-193.

  1. Macrodynamic Analysis
    1. The Nature of “Business Cycle” Theories.
    2. First-Order Difference Equations
      1. The Cobweb Theorem
      2. Lagging of Consumption or Investment by One Period
      3. Introduction of Disturbances
      4. A Dynamic “Keynesian” Model
    3. Models Involving Higher Order Difference Equations
      1. “Interactions between the ‘Multiplier’ and the ‘Acceleration Principle’”.
      2. Inventory decisions as related to changes in consumption or investment in Plant and Equipment.
    4. Problems of Prediction

Readings:

1.*—Paul A. Samuelson, “Interactions Between the Multiplier and the Principle of Acceleration”, included in Readings in Business Cycle Theory, 261-69.

2.—Mordecai Ezekiel, “The Cobweb Theorem”, included in Readings in Business Cycle Theory, 422-42.

3.—J. M. Clark, “Business Acceleration and the Law of Demand”, included in Readings in Business Cycle Theory.

4.—R. F. Harrod, The Trade Cycle, Chapter 2.

 

Source: Duke University. David M. Rubenstein Rare Book and Manuscript Library. Economists’ Papers Archive. Martin Bronfenbrenner Papers, Box 25, Folder “Macroeconomics, Problems & exercises. 1 of 2. 1961-70, n.d.”.

Image Source: Douglas Clement, “A Golden History” in Minnesota Economics (Fall 2006), p. 2.

Categories
Berkeley Chicago Columbia Economists Fields Oxford Socialism

Chicago. Nutter ranks Soviet economy experts in reply to Friedman, 1962

 

From the January 1962 exchange of letters between Milton Friedman and G. Warren Nutter transcribed below, we learn that the University of Chicago was interested in potentially hiring some academic expert on the Soviet economy. Friedman asked Nutter to rank three possible candidates of interest. Nutter did just that and threw in a fourth name.

Long before turning to the history of economics as my major research interest, I entered academic economics in the field of comparative economic systems. One of the candidates mentioned in the correspondence, Francis Seton, wrote a signed [!] positive referee report for my 1986 article in the Journal of Comparative Economics, “On Marxian value, exploitation, and the transformation problem: A geometric approach“, that I honestly regard as one of my pedagogical high-water marks. Another one of the 1962 candidates, Gregory Grossman, was one of the distinguished outside referees to whom I owe a debt of gratitude for helping me clear the tenure hurdle at the University of Houston. It is a real pleasure to be able to add his Berkeley memorial and picture to this post.

___________________

Gregory Grossman (1921-2014)
IN MEMORIAM by Gerard Roland

Gregory Grossman, born in July 1921 in Kyiv, Ukraine, passed away on August 14, 2014. Grossman was one of the world’s most highly reputed scholars of the Soviet economic system. He was considered a towering figure in the study of the Soviet economy. His scholarly work shaped the thinking of generations of scholars in the US and throughout the world.

In early 1923 his family fled post-Russian Revolution chaos and famine and took a month-long journey on the Trans-Siberian Railway to Harbin, Manchuria. After completing high school in 1937 in Tientsin, China, he boarded a Japanese ocean liner en route to attend UC Berkeley where he completed his B.S. and M.A., respectively in 1941 and 1943. During World War II, Grossman served as artillery observer with the 731st Field Artillery Battalion during the Battle of the Bulge and completed his war duty in Czechoslovakia. He received a PhD in economics from Harvard University in 1953. He was a faculty member of the Department of Economics at Berkeley from 1953 until his retirement in 1992.

Grossman was the author of several books and many highly influential articles. He made key contributions to the understanding of the Soviet economic system. In a classic article, “Notes for a Theory of the Command Economy” (Soviet Studies, 1963), he coined the concept of the “command economy” to characterize the central planning system, where production and investment were guided by the commands of the communist party elite and where managers at all levels of the planning system strove to implement the commands embodied in the plan targets. In such a system, prices and money play no active role and serve only as accounting units. In such a system, autonomy of agents must be curbed to favor the implementation of plan commands. As his former student, Pennsylvania State University professor Barry Ickes, has noted: “His formulation of the command economy hypothesis provided the framework used by scholars of several generations.”

In an equally famous article “The ‘Second Economy of the USSR” (Problems of Communism, 1977), he also coined the complementary concept of the “second economy.” Because of the imbalances and shortages inherent in a necessarily imperfect planning system, decentralized forms of market exchange, though illegal, were necessary to correct the allocative mistakes of the command system. Grossman worked with professor Vladimir Treml of Duke University and others to conduct more than a decade of research on all aspects of this second economy, gathering massive amounts of evidence based on interviews with emigres from the Soviet Union. He had garnered detailed evidence on the extent of the second economy and on prices of goods and services in various locations of the USSR.

Grossman’s analysis of the Soviet economic system proved extraordinarily prescient. Over time, as the economic system became more complex, the second economy tended to expand and corrode the command system, which eventually collapsed while managers of state-owned enterprises appropriated the assets they controlled in a process of spontaneous privatization. This was the starting point of the transition to the market economy that was studied by the next generation of scholars.

Grossman was awarded in 1991 a lifetime achievement award from the American Association for the Advancement of Slavic Studies. Citing Grossman’s works on the “command economy” and the “second economy,” the award also noted his earlier, path-breaking book, Soviet Statistics of Physical Output of Industrial Commodities (1960), saying that the book “provided the profession with basic rules for working with distorted Soviet economic statistics and avoiding the many pitfalls of that enterprise.”

A colleague at Berkeley, Benjamin Ward, said there was a period in the Cold War of maybe 20 years in which Grossman “was the most knowledgeable person in the world about the Soviet economy.”

Grossman was an appreciated teacher. For decades, he taught the main undergraduate course on the Soviet economic system. He also supervised throughout his career a great number of graduate students who later became themselves well-known scholars of Eastern European economies.

Grossman was a polymath who had a deep understanding of the political, ideological, social and cultural underpinnings of economic life in the Soviet Union. As a result, he was widely sought out by his peers for comments on their scholarship. He was also known to be a consummate gentleman. He remained calm and composed in all circumstances and was known for his great sense of humor and generosity.

Family members said that, while he traveled widely, he had a particular love for Berkeley and the Bay Area’s lifestyle, culture, beautiful vistas and good weather.

In 1952 he married Cynthia Green and they had two children, Joel Grossman of Kuala Lumpur, Malaysia, and Amy Di Costanzo of Berkeley, California. In 1972, he married Joan Delaney, a UC Berkeley professor of Slavic Studies who stayed by his side until his death. He is survived by her; by his two children, six grandchildren and one great granddaughter.

Source: Senate of the University of California, Berkeley.

___________________

Francis Seton (Guardian obituary)

Francis Seton
An economist of ideal prices
By Maurice Scott

He was born Franz Szedo in Vienna, in the wake of the collapse of the Austro-Hungarian empire after the first world war. He was an only child; his parents had been born in Hungary, and were then citizens of Austria and had converted from Judaism to Christianity. His father ran a paper processing business in central Vienna, and Francis was educated there until 1938, when the Nazis were moving to annex Austria.

His interests lay in music and foreign languages, the latter taking him on visits to France and Britain. His parents, concerned at the Nazi threat, thought he should complete his studies abroad, and Francis contacted Balliol College, Oxford, when visiting England in 1937.

In March 1938, Germany invaded Austria. His father managed to arrange for Francis to go at once to London. Soon after, his parents also left Austria and Francis lost touch with them, fearing that they could be dead. But this story has a happy ending. In 1946 he learned that they had survived in Hungary.

From 1938 Francis read politics, philosophy and economics at Balliol, but by summer 1940 paranoia was widespread and he was classified as an enemy alien, albeit in category C, for those considered to pose the least danger. He was shipped to Canada in dreadful conditions.

By 1941 he was given the choice of freedom in Canada or return to Britain. As he wanted to fight the Nazis, he volunteered for His Majesty’s forces. Being still classed an enemy alien, he was allowed to join only the dogsbody Pioneer Corps. He met other aliens, including Arthur Koestler, Robert Maxwell and, most notably, a Russian soldier, who fired his interest in the language and the country.

By 1942, Francis was able to transfer to the Somerset Light Infantry, on detachment to Bicester. There, in spare moments he studied for an Oxford degree in Russian language and literature, helped by a refugee from the Bolshevik revolution who was at St Hugh’s, and this led, in 1946, to first class honours. In 1942, having been rejected on medical grounds as a glider pilot, his flair for languages led to a transfer to the Intelligence Corps.

In 1948, back at Balliol, Francis finally graduated with a first in PPE and became a British subject, having changed his name earlier. He was awarded a state studentship, to study the Soviet economy, the subject of his doctoral thesis. In 1950, he was elected to a Nuffield College research fellowship, followed by an official fellowship in 1953. He moved on from his interest in the Soviet Union to other countries in the developing world, and travelled widely. Eventually he became senior fellow, and took the lead in the election of two of Nuffield’s wardens.

Francis was immensely talented. His English literary style was a delight. He was multilingual, poetic, musical, and could play the piano with brilliance. For all this, and above all for his humour and friendship, he will be remembered.

He is survived by his wife, three children and nine grandchildren.

Francis Seton (Franz Szedo), economist, born January 29 1920; died January 7 2002.

Source:  The Guardian, March 21, 2002.

___________________

BIOGRAPHICAL NOTE: Alexander Erlich

Alexander Erlich was born in St. Petersburg, Russia, in 1912. In 1918, shortly after the outbreak of the Bolshevik Revolution, his family immigrated to Poland where his father, Henryk, became a leader of the Jewish Labor Fund. After the execution of his father in 1941, Erlich and his family fled to the United States. Influenced by his father’s work and the political atmosphere of his youth, Erlich began his study of economics at Friedrich-Wilhelm University in Berlin and the Free Polish University in Warsaw. He completed these studies after moving to the US, earning his PhD from the New School for Social Research in New York City in 1953. His doctoral dissertation, The Soviet Industrialization Controversy, was the basis for his best known work, The Soviet Industrial Debate, 1924-1928, published in 1960. His lifelong devotion to the study of Soviet economic conditions and policies found Erlich a home at Columbia University. Beginning as a visiting lecturer in 1955, he received a tenured position as professor in 1959. He retired in 1981 only to return as a part-time lecturer and professor at Columbia University and Barnard College in 1982. Erlich died of a heart attack in January 1985 at the age of 72.

Source: Columbia University Archival Collections. Alexander Erlich papers, 1953-1985.

___________________

Obituary of Eugène Zaleski (1918-2001)

Slavic Review 61, no. 3 (Fall 2002), 681-682.

___________________

Arcadius Kahan (1920-1982)

After his arrival in the United States he earned a Masters in 1954 and Ph.D. in 1958 in Economics from Rutgers University.

He joined the Economics faculty at the University of Chicago in 1955. As a member of the Economics Department at the University of Chicago, Kahan straddled a fine line between the principles which he brought from his socialist youth and the neoclassical school of economic thought associated with the Department. He won the confidence of Milton Friedman with his work on the economic effects of the persecution of Jews in 19th century Russia. Kahan concluded that this had a significant impact on Russia’s economic backwardness, particularly as compared with western Europe. He argued that this was an example of dysfunctional governmental interference in the economy, which drew on the methodology of the neoliberals in the Chicago school.

Source: Arcadius Kahan, Wikipedia.

___________________

Carbon Copy of Letter
from Friedman to Nutter

January 16, 1962

Professor G. Warren Nutter
Department of Economics
University of Virginia
Charlottesville, Virginia

Dear Warren:

There is again some talk around here of getting a Russian expert and various names have come up in the discussion. Three names that seem to stand out are Seton, Grossman, and Alex Ehrlich [sic]. I wonder if I could impose on you to send me a brief and frank note on these three people in terms of their scientific capacities in general as well as their special competence in the Russian field.

As you may know, what is involved here is part of a broader program than one that the Department alone is involved in. I have no special responsibility for this and am just writing as a member of the Department.

I do not know what has happened with respect to Kahan. I know that the College here has proposed making him a permanent tenure offer. The Department while expressing concurrence in this has not been willing to make this a joint appointment. I know neither whether the appointment has been approved by central administration nor whether Kahan has accepted it. Needless to say, this is all highly confidential.

Trust things are looking up for the Center. Best regard and wishes.

Sincerely yours,

Milton Friedman

MF:mp

*  *  *  *  *  *  *  *  *  *  *  *  *  *  *

Nutter’s Reply to Friedman

University of Virginia
James Wilson Department of Economics
University Station
Charlottesville, Virginia

January 24, 1962

Professor Milton Friedman
Department of Economics
University of Chicago
Chicago 37, Illinois

Dear Milton:

I am glad to give my opinion on Seton, Grossman, and Erlich if it can be of help in the current deliberations of your department. I can indicate at the start that I consider Grossman to be the best of the three alternatives for reasons that will emerge from my comments.

I know Seton from his work, from listening to papers he read in England, and from various personal contacts with him. Seton writes with a lucid and interesting style as so many scholars trained in England do, but as is so often also the case the content does not measure up to the form. Most of his work, both analytical and empirical, seems to me to be quite superficial. As far as I know, he has not yet done a single piece of really serious research on an important problem. His one effort in the field of measuring industrial production has, in my opinion, received far mor attention than it deserves, aside from being wrong and misleading. In brief, I believe Seton still has to prove himself an original scholar of depth.

This cannot be said of Alex Erlich, whose work I know firsthand from his participation in the early stages in the N.B.E.R. project. Erlich has done some very creditable research, resulting in one book (his doctoral dissertation) and joint authorship of several other research papers of varying length. His major weakness on the empirical side is that he is somewhat slow and lazy, requiring continuous prodding to get work done. It is for this reason that most of his work has been done under somebody’s supervision. He has considerable difficulty in expressing himself orally, speaking very slowly and haltingly, but this does not carry over at all into his written work, which is generally clear and precise. Finally, he is weak and poorly trained on the theoretical side.

Grossman is clearly the most able economist in this group, and in addition he expresses himself extremely well. If anything, like Seton, he writes too well, being tempted to substitute pen and paper for thorough research. The only solid piece of research that he has done so far is the book that he wrote for us in the N.B.E.R. project. At the same time, he must be recognized as an able technician, thoroughly versed in economic theory and capable of making important contributions in the field of Soviet studies. The only problem to date is that he has not fully lived up to promise.

I should say that all three men are highly knowledgeable as far as detailed workings of the Soviet system are concerned, Erlich and Grossman probably more so than Seton. They are all three very agreeable and cooperative persons and would fit in well with any group of first-rate economists.

There is one person, less well known that the three you are considering but in my opinion very able, whom you should consider for this position. He is Eugene Zaleski, a Pole by birth but now a French citizen. While not an outstanding theoretical economist, he is the soundest person I know among Soviet specialists in interpretations of the working of the Soviet system. He is currently working on a long-range project on the Soviet planning mechanism and the relation between plan and outcome, the first volume of his work being scheduled to appear shortly. Unfortunately, he has been caught up in the French research apparatus with all the inevitable handicaps on successful individual research. Given the right opportunity, I feel that Zaleski could develop into an outstanding scholar in the field of Soviet studies. Among other things, he has a very quick and receptive mind, and he is a pleasure to work with.

I hope these brief comments will be of some use to you. To repeat, I think Grossman would be the best bet of the three persons you mentioned.

As to the Center, things are definitely looking up. We have already received since the conference $25,000 in essentially unrestricted grants, and the Lilly Endowment was most cordial and receptive to my pleadings and probably will contribute something.

Cordially,
[signed] Warren
G. Warren Nutter

GWN:jas

 

Source: The Hoover Institution Archives. Papers of Milton Friedman, Box 31, Folder 16 “Nutter, G. Warren.”

Image Source:  Gregory Grossman, Authority on Soviet Economy, Gregory Grossman, Passes Away, UC Berkeley News. August 25, 2014.

Categories
Chicago Economists Exam Questions

Chicago. Economic Price Theory Prelim Exam taken by Zvi Griliches. Winter quarter 1955.

 

With this post Economics in the Rear-view Mirror adds two more preliminary exams from the University of Chicago (here, from the Winter Quarter of 1955) to its growing collection of artifacts that provide us a digital record of economics education through the years. The original document was found in Milton Friedman’s files which provide us the additional information of the names of the examination committee as well as names, together with Friedman’s own test scores and his answers to the True-False questions. Of interest to note: Zvi Griliches not only attained the greatest number of points awarded by Friedman (120 points of 185 possible points), but he finished far ahead of the rest of the pack–the second highest exam only received 86 points which incidentally was more than enough to clear this PhD requirement. The Committee failed two students and four students passed the exam for the M.A. degree. Milton Friedman appears to have been the toughest grader of the three members of the Committee.

_____________________________________

Economic Theory Examination Committee:
M. Friedman, chairman; F. H. Knight; D. G. Johnson.

There were 13 examinees for Economic Theory I. These included Zvi Griliches (who incidentally blew the top off the curve according to Friedman’s grades) and Walter Oi.

Griliches Interview with Alan Krueger and Timothy Taylor from June 21, 1999.
Memorial blogpost for Walter Oi by Steve Landsburg on December 26, 2013

There were 2 examinees for Economic Theory II.

_____________________________________

Previously transcribed and posted Preliminary and Field Exams
from the graduate program of the University of Chicago

Economic Theory I and II. Summer 1949
Economic Theory I and II. Summer 1951
Economic Theory I and II. Summer 1952
Economic Theory I. Summer 1955
Economic Theory I and II. Winter 1955
Money and Banking. Summer 1956
Economic Theory. Winter 1957
Money and Banking. Summer 1959
Economic Theory (Old Rules). Summer 1960
Price Theory. Winter 1964
Income, Employment and Price Level. Summer 1967
Money and Banking. Summer 1967
Price Theory. Winter 1969
Income, Employment, Price Level. Winter 1969
Money and Banking. Winter 1969
International Trade. Winter 1970
History of Economic Thought. Summer 1974
Price Theory. Summer 1975
Industrial Organization. Spring 1977
History of Economic Thought. Summer 1989

_____________________________________

Economic Theory I
Preliminary Examination
Winter, 1955
[Milton Friedman’s answers in square brackets]

Time: 4 hours.

Write your number and not your name on your examination paper. Please be brief in your replies.

  1. (30 points) Indicate whether each of the following statements is True, False, or Uncertain and justify your answer briefly.
    1. [False] Production of a commodity occurs under conditions of fixed proportions. The supply curve for A shifts to the right. It is to the advantage of the owners of A that expenditure on A shall have represented a small part of total costs.
    2. [False] A firm will not carry on production at a given level of output, if one factor exhibits increasing average returns at that output level.
    3. [appears to be False with True crossed out] When a firm is in equilibrium, the ratio of the price of a factor to the marginal physical product of the factor determines the marginal cost of production.
    4. [True or Uncertain] If the demand for output is perfectly elastic, a decline in the price of factor A will always increase the demand for factor B unless A and B are perfect substitutes (only two factors employed).
    5. [True] If the demand for output is less than perfectly elastic, a decline in the price of A may either increase or decrease the demand for factor B.
    6. [False] If a monopsonist is not a monopolist, it is possible to construct the monopsonist’s demand curve for a factor.
    7. [False] If all the factors used by a firm are paid the value of their marginal products, the sum of the payments will equal the total receipts of the firm.
    8. [False] If all factors are paid the value of their marginal products, it would not be possible to increase total real output of the economy by any change in the allocation of factors.
  2. (15 points) In an article on the British tobacco industry, the Economist remarked:
    “Since 1938 the industry has had to contend with a sixfold rise in the standard rate of tobacco duty, and a three- to fourfold increase in the average cost of its principal raw material—this includes the higher cost of dollar leaf bought since sterling devaluation. All eight duty increases have been automatically passed on to the smoker, but if duty is left out of account the increase in cigarette prices since 1938 has been no more than about 85 per cent.”
    What do you take “passed on” to mean in this sentence? What is its relation to the economic concept of “incidence”? What inference, if any, would you draw about the latter?
  3. (20 points) Assuming that a monopolist always fixes price so as to maximize profits, can the price of a commodity ever be lower when it is monopolized than when it is competitively produced?
  4. (30 points) Trace the development of the theory of consumer choice. Include in your answer an explanation of (a) the meaning attached by Smith to “effectual demand”, (b) the role assigned by Ricardo to demand in determining prices; (c) Jevons “the final degree of utility determines price”; (d) the contribution of Edgeworth, Fisher, and Pareto.
  5. (20 points) It is widely asserted that workers have less “bargaining power” than employers because there are more workers than employers. Discuss.
  6. (25 points) Discuss the following concepts (a) the “postponement” of consumption said to be involved in saving and investment, (b) “abstinence”, (c) “time preference”, (d) the “marginal efficiency of investment”, (e) the “marginal efficiency of capital”.
  7. (45 points) For each of the following methods of financing radio and television programs, indicate how the resulting structure of programs differs from the optimum: and under what conditions, if any, it would be an optimum. In interpreting “optimum”, assume that the only consideration is direct private benefit from the programs; neglect distributional effects, i.e., treat it as a purely allocative problem; and assume that there are no such public issues involved as “education” or “indoctrination”. On the technical side, assume throughout that there are a narrowly limited total number of frequencies or channels available in any one area. Make your answer as definite as possible in terms of the kind of people whose tastes are or are not catered to appropriately, the kinds of programs that are too numerous or too sparse, etc. In answering the question, assume throughout that it is possible without cost to know exactly the number and kind of people who listen to each program.
    1. The existing U.S. method of selling time to advertisers.
    2. Imposition of an annual license tax or fee on each set; auctioning off of time to private program producers; compensation of these producers by giving to each a share of the total tax collection equal to the fraction of total listener time devoted to his programs. Assume that advertising is forbidden.
    3. Some mechanical method whereby a subscriber can receive a particular program only if he pays through a coin-box arrangement for that particular program. The programs are to be provided by private producers who receive the payments, who buy time on the stations, as in the preceding case, and who can determine the amount charged for the programs they produce. Once again, assume that advertising is forbidden.

 

 

ECONOMIC THEORY II
Preliminary Examination
WINTER 1955

Time: 2½ hours.

Note: Write only your number, not your name, on your examination paper.

Answer question 1, and two others.

 

  1. Using the Table below, explain the variations in the real income, the price level, the velocity of circulation, the government and private investment, the rate of unemployment, the ratio of savings to income, and whatever else you consider significant.

TABLE

The following figures are based on the Economic Report to the President, 1955.
Note: (a) All figures except those for item A are expressed as percentages of the corresponding 1937 figure; (b) item F is defined to be equal to “gross private domestic investment” plus “government purchase of goods and services” plus“net foreign investment”, all in 1947 prices.

1929

1933

1937 1941 1945 1949

1953

A. Unemployment as percentage of civilian labor force

3.2

24.9 14.3 9.9 1.9 5.0

2.5

B. Civilian employment 103

84

100 109 114 127

134

C. Demand deposits and currency (non-deflated) 89

67

100 164 346 376

441

D. National income (non-deflated) 119

55

100 142 246 294

414

E. Consumer price index 119

90

100 102 125 166

186

F. Gross national product less consumption (in 1947 prices) 100

41

100 160 281 165

262

G.  D/C 134

82

100 87 71 78

94

H.  D/E 100

61

100 139 197 178

222

I.  H/B 97

72

100 128 172 140

166

J.  F/H 100

67

100 115 146 93

118

  1. It is often said that the U.S. economy is less likely to suffer a severe depression today than it was twenty or thirty years ago. List and discuss major changes which have taken place which bear on this statement.
  2. Suppose the tax on capital income (dividends, interest) is increased. What will be the effect on the demand for cash if the tax proceeds are spent on: (a) aid to foreign countries; (b) federal contribution to medical aid in the United States.
  3. In the Confederate States, the ratio of bank reserves to deposits grew rapidly during 1862-64. This ratio also grew in the period 1933-37 in the Unites States. Explain these phenomena. Evaluate the action taken by the Governors of the Federal Reserve Board in 1936 and 1937, when they raised the required minimum reserve ratio.
  4. The stock of money (currency and demand deposits) per capita was about 800 dollars in June 1953 as against about 100 dollars in June 1910. Explain the increase.

Source: Hoover Institution Archives. Milton Friedman Papers. Box 76. Folder 2 “University of Chicago, Economic Theory”.

Image Source:  Zvi Griliches. University of Chicago Photographic Archive, apf1-06565, Special Collections Research Center, University of Chicago Library.

Categories
Cambridge Chicago Economists LGBTQ Northwestern

Chicago. Economics Ph.D. alumnus, “gay godfather” and mentor. Roger Weiss, 1955

Milton Friedman wrote a recommendation for two University of Chicago economics graduate students to receive fellowships from the Earhart Foundation in 1953. Friedman’s letter was transcribed for the previous post that focussed on Gary Becker, who was the unambiguous first choice in Friedman’s eyes. In addition to adding to our stock of economics Ph.D. alumna/us stories, Economics in the Rear-View Mirror introduces the LGBTQ label here with Friedman’s second candidate for an Earhart Foundation fellowship, Roger William Weiss (Chicago, Ph.D., 1955). 

_____________________

Roger William Weiss. (1930-1991) Dissertation “Exchange Control in Britain, 1939-1952”, Ph.D. awarded Winter Quarter 1955.

_____________________

AEA Profile from 1969

WEISS, Roger William, academic; b. Bronxville, N.Y., 1930 stud., Northwestern U., 1946-48; M.A., U. Chicago, 1951, Ph.D. 1955; stud., Cambridge U., Eng., 1951-52. COC.DIS. “The British Exchange Controls, 1939-52,” 1954. PUB. “Economic Nationalism in Britain in the Nineteenth Century” (H.G. Johnson, Ed.), Econ. Nationalism in Old and New States, 1967; The Economic System, 1969; “The Case for Federal Meat Inspection Examined,” Jour. Of Law and Econs., Oct. 1964. RES. American Colonial Monetary System. Asst. prof., Vanderbilt U., 1953-57; pres., N. Weiss & Co., Inc., 1957-63; asso. Prof., U. Chicago since 1966. ADDRESS 1415 E. 54th St., Chicago, IL 60615.

Source: The American Economic Review, Vol. 59, No. 6, 1969 Handbook of the American Economic Association (Jan., 1970), p. 467.

_____________________

U. of Chicago obit for Roger W. Weiss

Roger Weiss, AM’51, PhD’55, professor in the social sciences since 1963, died March 7. His specialty was the role of economics in the arts and the international trade of art works. His books included The Economic System and The Weissburgs: A Social History, a history of his own family. He was also a member of the governing board of the Chicago Symphony Orchestra. Survivors include his mother, Irene, and a brother, John.

Source: University of Chicago Magazine, Vol. 83, No. 5, June 1991, p. 44.

_____________________

Roger Weiss and his partner, Howard Brown, in the University of Chicago gay community

Roger Weiss AM 1951, PhD 1955. Professor in the College and division of social sciences. Partner Howard Mayer Brown (1930-1993), Ferdinand Schevill distinguished service professor of music.

Professors Howard Brown and Roger Weiss were “out” by many standards. The University agreed to a “spousal hire” for the couple in the 1960s, and the two hosted parties for gay students and faculty in their home until Roger’s death in 1991, and Howard’s death in 1993. Bob Devendorf (AB 1985, AM 2004) remembered Howard and Roger as “gay godfathers” and mentors, while John DelPeschio (AB 1972) treasured the intergenerational community they fostered: “I felt as if I were entering a more adult world.”

However, Brown and Weiss’ refusal to participate in political actions and “come out” in the broader public sphere sometimes frustrated younger gay men like Wayne Scott (AB 1986, AM 1989), as he describes in this article. Jim McDaniel (AB 1968) remembers Howard saying “I don’t really care what anybody knows, I just care what I have to admit.”

Source: Closeted/OUT in the Quadrangles. A History of LGBTQ Life at the University of Chicago

 

Image Source: Senior year picture of Roger W. Weiss from the 1946 Hyde Park High School Yearbook, The Aitchpe.

 

Categories
Chicago Economists

Chicago. Friedman recommends Becker and Weiss for Earhart Fellowships, 1953

 

The following letter of recommendation by Milton Friedman provides us a glimpse of the young Gary Becker. It is also interesting to observe the language used to describe potential superstardom as opposed to more conventional stardom in economics. The next post will provide career information for the “other candidate”, Roger Weiss.

______________________

THE UNIVERSITY OF CHICAGO
Chicago 37, Illinois
Department of Economics

January 27, 1953

Mr. James A. Kennedy
Earhart Foundation
First National Bank Building
Ann Arbor, Michigan

IN RE: BECKER, Gary S.

Dear Mr. Kennedy:

I am writing at the suggestion of Professors William Paton [University of Michigan] and John Van Sickle [Wabash College] to propose two young men for Earhart fellowships in economics: Gary Becker and Roger Weiss.

Gary Becker is a young man who received his A.B. from Princeton. He was recommended to us by his Princeton teachers for a departmental fellowship in terms that we found hard to take really seriously—the best person that we have had in the last ten years; the best student that I have ever had, and the like. After observing him closely for the past year and a half, I am inclined to use similar superlatives: there is no other student that I have known in my six years at Chicago who seems to me as good as Becker or as likely to become an important and outstanding economist. Though only twenty two years old now, Becker has already published one paper in the American Economic Review[*] and has collaborated with one of his teachers at Princeton in a paper published in Economica.[**] Both are first rate papers. Becker needs to do one more full year of graduate work to fulfill his requirements for his Ph.D. Our department has granted him a fellowship in the past and will again; in addition I believe he is applying for a Social Science Research Fellowship. I have asked him to summarize briefly his plans for next year, and enclose his brief statement. [not in this Hoover file]

Becker has a brilliant, analytical mind; great originality; knowledge of the history of economic thought and respect for its importance; a real feeling for the interrelationships between economic and political issues; and a profound understanding of both the operation of a price system and its importance as a protection of individual liberty. This is one of those cases in which there is just no question at all about Becker’s being preeminently qualified for one of your fellowships. I wish I could look forward to being able to find a candidate this good every year, but that is asking for too much.

Roger Weiss, the other candidate I would like to propose, is also an extremely able young man—he is not in Becker’s class, but that is a measure of Becker’s extraordinary qualities, not a reflection of Weiss. He is of the quality of the very top group of our graduate students.—the best half-dozen or so each year out of our 125 to 175 graduate students. He did some of his undergraduate work here; spent last year at Cambridge, England on a fellowship, and returned here this year for further graduate work. Another year should see him with his Ph.D. He has just turned twenty three.

Weiss has been working on a topic that he got interested in in England, namely, the operation of British Exchange controls in the post-war period. He came to the conclusion that their effectiveness was greatly overrated and their adverse effects on the efficiency of British industry greatly underrated. He is trying to see how far a more detailed study will support these judgments and permit them to be spelled out.

Weiss has an excellent mind and a thorough knowledge of price theory and monetary theory. His major interest is in problems connected with money and international trade. He is hardworking, conscientious, and productive. Perhaps his strongest quality is his ability to organize material well and to present it both in writing and speech lucidly and with some distinction. I expect Weiss to become a productive scholar and to have a most desirable influence through his writings on public policy. I have asked him, too, to prepare a brief statement of his plans, which I enclose. [not in this Hoover file]

I may say that I have checked these recommendations with my colleagues H. Gregg Lewis and Frank H. Knight, who concur in them.

Sincerely yours,
[signed]
Milton Friedman

MF-FF

[Handwritten:] P.S. This letter was written just prior to receiving yours of the 23rd. Both men do of course plan to go into University teaching.

[* “…taken from a larger essay originally submitted as a senior thesis in the department of economics and social institutions of Princeton University.” A Note on Multi-Country Trade. The American Economic Review, Vol. 42, No. 4 (Sep., 1952), pp. 558-568.]

[** The Classical Monetary Theory: The Outcome of the Discussion (with William J. Baumol). Economica, New Series, Vol. 19, No. 76 (Nov., 1952), pp. 355-376.]

Source:  Hoover Institution Archives. Milton Friedman Papers, Box 194, Folder “Earhart Foundation…”.

Image Source:  Becker-Friedman Institute for Economics at the University of Chicago. Webpage “About Our Legacy”.

Categories
Chicago Economists Exam Questions

Chicago. Preliminary Examinations in Economic Theory. Friedman, chair. 1952

 

Today’s post includes not only the questions for the economic theory preliminary examinations (Part I and Part II) from the summer quarter of 1952 at the University of Chicago, but also some interesting background material. From Milton Friedman’s papers at the Hoover Institution archives I have transcribed copies of the entire schedule of preliminary examinations for summer 1952 along with the correspondence between Friedman, Frank Knight and the departmental secretary. We can compare Friedman’s suggested questions with the questions that were actually used for the exam along with Friedman’s rankings of the anonymous examinations. Two sentences in Frank Knight’s letter to Friedman (after the grades had been compared among the graders and the veil of ignorance regarding the identities of the examinees was lifted) is definitely worth considering in light of current discussions about systemic elements of racism in the discipline of economics.

“I feel that these Negroes are in the same position as the Chinese students only more so in that they compete in a completely different market, and they are never really compared with our “full fledged” Ph.D. graduates. (Besides, between you and me, I have attended 4 or 5 Ph.D. exams this summer and thought very few of them ought to pass but they all did).”

I have gone on to track down the top eight examinees as ranked by Milton Friedman. Fun facts: Gary Becker won the bronze medal and Abba Lerner’s son, Lionel Lerner, placed fourth.

The summer 1951 theory preliminary exams were posted earlier.

_________________________

Schedule for the Preliminary Examinations
Summer 1952

July 15, 1952

To: Committee members of Preliminary examinations
From: J. Barker, Departmental Secretary
Re: Schedule and committees for Preliminary Examinations, Summer Quarter, 1952.

Date Examination Committee Registration
Tues., July 29 Economic Theory I M. Friedman, Chr.,
F. H. Knight
G. Tolley
26
Thurs., July 31 Economic Theory II (as above) 4
Tues., July 29 Government Finance P. Thomson, Chr.
H. Lewis
1
Thurs., July 31 Industrial Relations F. Harbison, Chr.
A. Rees
M. Reid
1
Tues., Aug. 5 Money, Banking & Monetary Policy L. Mints, Chr.
E. Hamilton
J. Marschak
21
Tues., Aug. 5 Statistics T. Koopmans, Chr.
W. Wallis
4
Thurs., Aug. 7 Agricultural Economics D. Johnson, Chr.
T. Schultz
P. Thomson
8
Thurs., Aug. 7 International Econoics L. Metzler, Chr.
C. Hildreth
H. Lewis
9

_________________________

Friedman to Knight and Tolley
Carbon copy

Orford, N.H.
July [19 or 20], 1952

F. H. Knight
G. Tolley

Dear Knight and Tolley:

I have just received word from Miss Barker that I am chairman of the Theory prelim committee for this summer, that you are the other members, and that the exams are to be in her hands by July 22.

I wish you could join me here for a session to get out the exams—and I am sure you do too if what we have been hearing about the weather in Chicago bears any resemblance to the truth.

Since you cannot, I enclose some suggested questions for both Part I and Part II. I wonder if the two of you could get together and combine these or such of them as you think worthy of retention with your own questions. Time does not permit of rechecking with me and I assure you I shall be more than satisfied with whatever decisions the two of you make.

As to the papers, have them sent to me at any stage that suits your own plans best, since mine are very flexible. I shall try to read them promptly and return them promptly. If I send you in my grades, perhaps the two of you can combine them with your own. I realize this puts more of the work on you, but I know not what else to do. I do hope we can get the grades in reasonably promptly, and certainly before the end of the quarter, which also means before I return.

Many thanks, and apologies. Best regards too.

Yours,

_________________________

Friedman’s proposed theory exam questions
Summer 1952

M. Friedman

Suggested Questions for Theory Prelim, Summer, 1952

Part I

  1. Define the following terms precisely and indicate briefly the use made of each in economics:
    1. Demand
    2. Supply
    3. Equilibrium
    4. Indifference Curve
    5. Marginal
    6. Rate of Substitution
    7. Marginal value product
    8. Marginal efficiency of capital
    9. Production function
    10. Time preference
    11. Profit
    12. Rent
    13. Run
    14. Net advantages
    15. Variable Costs
  2. (a) “I wouldn’t take it if you paid me”. Draw the consumption indifference curves implied by this statement. (You may find it helpful to suppose first that there is some finite minimum price per unit at which the speaker would take “it”; then approach the limit implied by the quotation.)
    (b) “I’ve reached the point of diminishing returns, so I better quit”. Analyze, indicating under what conditions and for what definition of diminishing returns this is a valid inference from the conditions for a maximum.
  3. (a) Complaints are often heard about the “high” incomes of bootleggers in dry states, or gamblers where gambling is illegal, or smugglers, etc. Are high incomes in such cases evidence of the success or the failure of the laws? Explain your answer.
    (b) A man buys a ticket in a lottery and wins. View this as a business transaction. How much, if any, of his prize is properly regarded as “profit”? Does your answer use the concept of “profit” implicit in the common statement “entrepreneurs seek to maximize profit”? Justify your answer and indicate the difference, if any, between the two concepts.
  4. (a) Outline the theory of joint supply
    (b) What factors determine the elasticity of the derived supply curve of one of a pair of jointly supplied items? Show the direction of influences and prove your statements graphically or otherwise.

*  *  *  *  *  *  *  *  *  *  *  *  *

M. Friedman

Suggested questions for theory prelim, Summer, 1952

Part II

  1. During every hyper-inflation there are always recurrent complaints of a “shortage of money.” How do you explain this phenomenon?
  2. The following quotation is from an article on the illicit gold traffic:
    “Traffic on the Asian gold-smuggling trails has doubled since Korea…Meanwhile savings which could be productively invested by banks lie idle; paper money is snubbed for gold, depreciates with every rise in the gold price, and becomes a weaker and weaker factor in national economies.” (H.R. Reinhardt, The Reporter, July 22, 1952, p.21).
    Analyze this quotation. Precisely what effect would the willingness of people to hold bank deposits instead of gold have on productivity or productive investment, and through what channels? What of sense and what of nonsense is there in the statements after the semi-colon?
  3. There has been much talk of the so-called “wage-price spiral.” What is generally meant by this term? Give a theoretical analysis of the so-called spiral, indicating under what circumstances you think it could or could not arise.

_________________________

Actual Economic Theory Preliminary Examination Questions
Summer, 1952

Summer, 1952

ECONOMIC THEORY I

Time: 4 hours

Answer all questions.

  1. Define the following terms precisely and indicate briefly the use made of each in economics:
    1. Demand
    2. Supply
    3. Indifference Curve
    4. Rate of Substitution
    5. Marginal value product
    6. Marginal efficiency of capital
    7. Production function
    8. Time preference
  2. (a) Outline the theory of joint supply
    (b) What factors determine the elasticity of the derived supply curve of one of a pair of jointly supplied items? Show the direction of influences and prove your statements graphically or otherwise.
  3. Assume that Crusoe is interested in economizing the use of his resources and that during the period in question there is no change in his knowledge of production techniques. How does capital and interest theory aid in explaining the following observations?

(a) After several years, Crusoe begins to obtain berries by planting and cultivation rather than simply by picking them as he had done previously.
(b) After an additional number of years, he reverts to picking wild berries.

  1. What theories do you offer to explain the following phenomena?

(a) During a prolonged rise in the general level of prices, the price of soft drinks remained at five cents with no change whatsoever in the physical characteristics of the product.
(b) During a prolonged rise in the general level of prices the price of candy bars remained at five cents, at the same time, however, as the size of the bars decreased.

  1. Using diagrams, briefly discuss the long-run cost curve for a competitive industry. Indicate, with diagrams, the response to be expected from (a) an expansion of demand, (b) a decrease of demand, within periods too short for a significant change in the fixed investment.
  2. Briefly state the main changes in the body of accepted price theory at the turn from “classical” to “Austrian” (the subjective-value school), i.e., at the “revolution” of the 1870’s. Similarly describe the transition from Austrian to “New-classical” (Marshallian) doctrine.

*  *  *  *  *  *  *  *  *  *  *  *  *

Summer, 1952

ECONOMIC THEORY II

Time: 2 ½ hours

Answer all questions.

  1. During every hyper-inflation there are recurrent complaints of a “shortage of money.” How do you explain this phenomenon? Compare the situation during acute depression.
  2. A part of the nation’s productive capacity is destroyed, say by a war. Ignoring any possible expectational and distributive effects, how will this affect: (a) the division of the national income between consumption and investment? and (b) the income-velocity of money. How, if at all, does your answer depend on whether wealth is a variable which influences behavior?
  3. There has been much talk of the so-called “wage-price spiral.” What is generally meant by this term? Give a theoretical analysis of the so-called spiral, indicating under what circumstances you think it would or would not arise.

_________________________

Theory Prelim, Summer, 1952, Part I. Grades by M. Friedman

General notes:

  1. I have classified the papers into five groups.

P—clear pass for the Ph.D. (7 papers)
P(?) Questionable pass for Ph.D. (5 papers)
A.M. Pass for a.M./questionable fail for Ph.D. (5 papers)
F(?) Questionable fail for A.M., clear fail for Ph.D. (4 papers)
F Clear fail for both (4 papers)

Should emphasize that as always this is somewhat arbitrary. In particular, difference between two fail classes is particularly small in this batch.

  1. In addition to the above class mark, Igive the ranking by my numerical grades. 1 is the best paper, 2, the next best, etc., to aid in seeing whether any differences among members of the committee reflect differences in absolute or relative grading.
# of candidate. Class grade Rank Remarks
1 AM 16
2 F 24
3 P 6
4 P(?) 8
5 P 5
6 F(?) 21
7 AM 14
8 P(?) 11
9 AM 15
10 P 4
11 P(?) 12
12 F 25
13 P 2 This and 15 distinctly the two best papers
14 F(?) 18
15 P 1 See under 13
16 AM 13
17 AM 17
18 F 23
19 F 22
20 P 7
21 P 3
23 F(?) 19
25 P(?) 10
26 F(?) 20
27 P(?) 9

 

PART II OF THEORY PRELIM

Not one of the three papers submitted on this part seems to me satisfactory. #1 is the best of the three, though not by much, and might deserve a questionable pass. Both of the others seem to me clear failures.

_________________________

 

THE UNIVERSITY OF CHICAGO
Chicago 37, Illinois
Department of Economics

September 8, 1952

Mr. Milton Friedman
Orford
New Hampshire

Dear Milton:

Tolley and I have just gone over our three reports and find them fairly well in agreement. The most serious exception is #7—John J. Klein, whose paper you marked passable for the A.M. only, while both Tolley and I gave him a clear pass. Your rank was 14, as you probably have the record to show. What do you suggest? It will be no great hardship to us to re-read the paper, and we shall do so with the next day or so. Do you want to see it again? Or what can we report?

Another questionable case is Adolph Scott (Colored). Here I am the odd man, as I marked him passable, while you ranked him 23 out of 25, and Tolley ranked him 24. I yield as far as passing him for the Ph.D. is concerned but wondered what you would think about passing him for the A.M. He seems to have squeezed through in International Trade at the A.M. level. This would allow him to get the Master’s degree. I feel that these Negroes are in the same position as the Chinese students only more so in that they compete in a completely different market, and they are never really compared with our “full fledged” Ph.D. graduates. (Besides, between you and me, I have attended 4 or 5 Ph.D. exams this summer and thought very few of them ought to pass but they all did).

On Part II there is also some discrepancy. I had Mints read these papers, and he and I agree that #2, Mrs. Mullady, was passable. But you and Tolley both wrote failure and as she failed “flat” on Part I and has also failed a second time in another field, it looks as though that disposes of her case. This leaves S. Smidt who has your vote, a questionable pass, Tolley’s a clear pass, and Mints and I though a very very [sic] dubious pass. But Smidt passes Part I with colors flying. I am perfectly willing and in fact disposed to yield on him and pass him as I don’t feel competent to grade these Part II papers anyway.

Cordially,

(Dictated but not read)
Frank H. Knight

Source: Hoover Institution Archives. Milton Friedman Papers. Box 76. Folder 2 “University of Chicago ‘Economic Theory’”.

_________________________

Identities of eight examinees given passing grades
by Milton Friedman by rank

First place

Seymour Smidt. University of Chicago Ph.D. (1954). Dissertation: “Efficient Management for Government Wheat Stocks”.

Second place

Conrad Jan (Coen) Oort. University of Chicago A.M. (1954). Doctor of Economics, University of Leiden (1958).

Professor economics, U. Utrecht, The Netherlands, 1960-1971; professor economics, University of Michigan, Ann Arbor, 1956-1957; treasurer-general, Treasury, The Hague, The Netherlands, 1971-1977; managing director, Algemene Bank Nederland Bank (now Algemene Bank Nederland-AMRO), Amsterdam, The Netherlands, 1977-1989; non-executive director various companies, The Netherlands, since 1989; professor economics, Maastricht, The Netherlands, since 1986. Chairman KLM, Amstelveen, Netherlands, 1992, Robeco Group, Rotterdam, Netherlands, 1989. Vice chairman Aegon Insurance, The Hague, 1990.
Source: Prabook webpage for Conrad Jan Oort.

Third place

Gary S. Becker. University of Chicago Ph.D. (1953). Dissertation: “The Economics of Racial Discrimination”.
The Sveriges Riksbank Prize in Economic Sciences in Memory of Alfred Nobel 1992.

Fourth place

Lionel John Lerner. [son of Abba P. Lerner and Alice Sendak]. University of Chicago A.B. (1950) and A.M. (1952). Johns Hopkins University Ph.D. (1955). Dissertation: “Theories of Imperialist Exploitation.”
Source: Johns Hopkins University, Sheridan Libraries, Special Collections. Commencement Program 1955, p. 19.

Fifth place

Edward J. Kilberg. Hofstra University B.A. (1949). Duke University A.M. (1952). University of Chicago A.M. (1957).
Apparently Kilberg was never awarded a Ph.D. in economics by the University of Chicago for his dissertation “Commercial bank holdings of cash and liquid items”. Most likely reason is that he died in the crash of a Northeast airliner at Nantucket Airport on August 15, 1958. Kilberg left a research job at the Mutual Life Insurance Company in 1957 to go to the NBER where he worked as assistant to Arthur F. Burns for the book Prosperity Without Inflation (1958).

Sixth place

Hugh Roy Elliott. In the list of economics Ph.D. dissertations kept by the department of economics at the University of Chicago we find “Hugh R. Elliott. Dissertation: Savings Deposits as Money (Summer 1964)” which seems rather late in the game. But then we see: AER Sept. 1957, p. 838 “Hugy [sic] R. Elliott, B.A. Harvard 1950; M.A. Chicago 1952.” Thesis in preparation at Chicago “Savings deposits as money”.

Seventh place

Irwin Ira Baskind. I have found the following item “Baskind, Irwin. Postwar Monetary Policy in Belgium (Ph.D., Chicago)” from U.S. State Department, Bureau of Intelligence and Research. External Research. A List of Studies Currently in Progress, Western Europe, ER list no. 5.14 (April 1960), p. 9. Note: Baskind’s name does not appear in the list of economics Ph.D.’s kept by the Chicago department of economics.

Eighth Place

Paul Gabriel Keat. Baruch School of the City University of New York B.B.A. (1949). Washington University A.M. (1950). University of Chicago A.M. (1952, 1956). University of Chicago Ph.D. (1959). Dissertation: “Changes in Occupational Wage Structure 1900-1956”.

Keat, Paul G. PhD 88, passed away on April 2, 2014.Born in Prague, Czechoslovakia May 2, 1925. A WWII vet who served in Ardennes, Normandy and Rhineland. Decorated with the European African Middle Eastern Services Medal, Good Conduct Medal and WWII Victory Medal. Discharged 1946. Graduated 1959 from the University of Chicago with an M.A. and PhD in economics. Student of his cherished professor, Dr. Milton Friedman. Earned B.B.A. in accounting from Baruch School of the City University of New York and M.A. from Washington University. Paul’s work with IBM was extensive in both the United States and in the European headquarters based in Paris. He taught both finance and economics at the graduate level in numerous universities including Syracuse University, Washington University, the City University of New York, Iona College and the Lubin Graduate School of Business at Pace University. In 2013 he co-authored and published the seventh edition of his textbook “Managerial Economics”.
Source: Arizona Republic, Phoenix. April 13, p. F9.

Images: The economic theory prelim examiners, Friedman, Knight, and Tolley. From the University of Chicago Photographic Archive.

Categories
Chicago Exam Questions

Chicago. Graduate Preliminary Examination, Money and Banking, 1967

 

This copy of the 1967 Money and Banking prelim exam comes from Milton Friedman’s papers and has Milton Friedman’s name noted. So we may strongly presume that Friedman was in fact on the Money and Banking prelim committee as he was on the Income, Employment, and Price Level prelim committee that year.

______________________

Previous posts with University of Chicago preliminary examinations for Ph.D. and A.M.  degrees:

Preliminary Exam (Economic Theory I) 1955

Preliminary Exam (Money and Banking) 1956

Preliminary Exam (Economic Theory) 1957

Preliminary Exam (Money and Banking) 1959

Preliminary Exam (Economic Theory, Old Rules) 1960

Preliminary Exam (Price Theory) 1964

Preliminary Exam (Income, Employment and Price Level) 1967

Preliminary Exam (Price Theory) 1969

Preliminary Exam (Macroeconomics) 1969

Preliminary Exam (Money and Banking) 1969

Preliminary Exam (International Trade) 1970

Preliminary Exam (Price Theory) 1975

Preliminary Exam (Industrial Organization) 1977

Preliminary Exam (History of Economic Thought) 1989

___________________________

[Handwritten note, top of page: “Mr Friedman”]

MONEY AND BANKING
Preliminary Examination for the Ph.D. and A.M. Degrees
Summer, 1967

WRITE THE FOLLOWING INFORMATION ON YOUR EXAMINATION PAPER

—Your code number and NOT your name
—Name of examination
—Date of examination

Results of the examination will be sent to you by letter.

ANSWER ALL QUESTIONS—ALL QUESTIONS HAVE EQUAL WEIGHT

1. a) “The fallacy in the quantity theory of money is that it allows for the circulation of money but not the circulation of goods. A correct theory would have a velocity of circulation of goods to parallel the velocity of circulation of money.” Discuss.

b) According to one writer, one of the “fundamental laws of economics” is that “the inflation rate is approximately equal to the interest rate when averaged over several decades.”
(Andre Gleyzal, “Theory of Money in a Free Economic System.” Discuss (and do not dismiss out of hand).

2. a) What is the “Phillips Curve”?

b) Give the theoretical analysis on which it rests. Do you regard it as valid? If so, defend it; if not, why not?

c) What is its relation to the notion of a “trade-off” between unemployment and inflation?

d) What is your understanding of the present state of the empirical evidence on the Phillips curve?

3. a) Expand the standard analysis of the IS-LM (or EEL) curves to include foreign trade and the balance of payments when all economies are operating with fixed exchange rates under a pure gold standard.

b) Would this analysis be any different under

i) fixed exchange rates with national currency standards?
ii) floating exchange rates?

Why, or why not?

4. a) A once and for all change in the money supply is expected to affect only the price level and not any real economic magnitudes. Yet some economic theorists who accept the neutrality of money in this sense argue that a sudden decrease (say) in the money supply will cause unemployment. How do you reconcile these two positions?

b) Assume that a country is operating on a classical gold standard. It has a central bank but the bank does not engage in open market operations. It confines its policy to setting an interest rate (discount rate) at which it lends freely. Let important gold discoveries be made in that country such that, at the prevailing price of gold, the rate of gold production increases. Does the neutrality of money still hold true in the long run? Will the increased rate of gold production affect only the price level and not the level of real income in the given country?

5. Most empirical studies of the demand for money that use time series data take the real stock of money as the dependent variable and take measures of real income or wealth and of the interest rate as explanatory variables. However, most monetary theorists treat the nominal stock of money as exogeneous. This appears inconsistent with the empirical work. Can you describe a sensible economic model to defend the choice made by the empirical investigators? Assume it is your purpose to predict the increase in the demand for real money balances resulting from an increase in real income. For simplicity, assume that current real measured income is the relevant income variable. Do not discuss the econometric theory of identification, etc. Focus your attention on the economic hypotheses in terms of the price level, the nominal money stock, interest rates, and nominal income. Would it be better to treat real money balances as an explanatory instead of as a dependent variable in estimating the demand for money?

6. Comment on the following proposition:

In the portfolios of banks, private loans and government bonds are alternatives. The smaller the quantity of loans that banks make (i.e., the tighter the supply of bank credit), the greater must be the quantity of government bonds the banks are holding in their portfolios. But the total supply of government bonds is fixed, and so this implies that the tighter is bank credit, the smaller the supply of government bonds available to the non-bank public to hold in their portfolios. But the smaller the quantity of government bonds available to the non-bank public, the greater the quantity of other assets they will hold. In other words, the tighter is bank credit, the greater the supply of private credit from non-bank holders of wealth, and the portfolio behavior of banks is largely irrelevant in determining the total supply of private credit.

Source:  Hoover Institution Archives. Papers of Milton Friedman. Box 77, Folder 8 “University of Chicago Econ. 331”.

Image Source:  “Money Talks” from the cover of Puck, Vol LX, No. 1541 (September 12, 1906). Library of Congress Prints and Photographs Division Washington, D.C.  “William Randolph Hearst sitting with two large, animated, money bags resting on his lap, with arms and legs, and showing two large coins as heads; on the floor next to Hearst is a box labeled ‘WRH Ventriloquist’.”